For answer choice A, I seemed to struggle understanding how it strengthens the argument. Is answer choice A saying that the more "good" cases that you have, the higher your productivity score at your firm (I am assuming that a high productivity score is a ...
Can someone help explain the reasoning for the correct answer? I'm having a tough time sorting through this one. It seems like Antoine's response to Giselle consists of 2 separate arguments: 1) the gas tax shouldn't be increased because its unfair. 2) If ...
In the question, the advertisement claims no headache pill stops pain more quickly. Then, to my undestanding, their claim is that their pill, compared to their competitors, stops pain the quickest. Why is the answer C as opposed to D?
I thought B and C both worked. Wouldn't the first sentence explain why the public is so concerned about water issues when pollution is just as serious an issue?
As for AC A, I think I understand that AC A meets the target by introducing a third party, stress, that could be responsible for causing both snoring and smoking. However, I am wondering that what if stress ...
This is a necessary assumption question. The correct answer : "parts that satisfy our govt standards are not the same poor quality as cheap foreign parts." But the premise already states that the country has the toughest automotive tests in the world. I ...
I don't understand the correct answer at all. I don't see how it could possibly be correct. I chose C which looked like the absolute only correct option.
Can someone please share some knowledge with me revolving around question 12 of PT.33? My specific question pertains to understanding the written logic, which is used to solve the question.
I don't understand how you would go about this question because all of the answer choices sound bad. Why would A be the answer? Is it because you're pointing out a potential alternate cause?
I'm confused on what ac B is trying to convey. Is it trying to state that he should not have inferred that just because some people did not like the segment, others must have OR is it stating that he should have inferred but did not that since some did not ...
"not a problem in well-ventilated house but it is in well-insulated house" implies that a well-insulated house is not a well-ventilated house, hence D, correct? I picked E because I mistakenly read the well-ventilated as well-insulated in that a well- ...
So phenomenon - El Niño - is expected to increase in coming years and this thing causes HEAVY WINTER RAINFALL in T.
Conclusion: Average rodent population in T will increase in the coming years.
WHY: Because rodent populations normally ...
Can someone explain why E and not B is the correct answer? I am not totally convinced by the explanation given [here](https://7sage.com/lsat_explanations/lsat-147-section-4-question-12/). This is because the idea that the work is split equally is not ...